7

Here is my question :

Let $n$ a squarefree positive integer, $m \ge 2$ an integer and $a+b \sqrt n \in\mathbb Q (\sqrt n).$ What (sufficient or necessary) conditions should $a$ and $b$ satisfy so that $a+b \sqrt n$ has a $m$-th root in $\mathbb Q (\sqrt n)$?

Here is my attempt : I tried the case $m=2$. If $\sqrt{a+b \sqrt n} = c+d\sqrt n$ with $c,d \in \mathbb Q$ then $$ a=c^2+d^2n, b=2cd. $$ Assuming $b \neq 0$, I get $c^2 + n\left(\frac{b}{2c}\right)^2 = a$, and for instance $c = \pm \sqrt{\frac{a+\sqrt{a^2-nb^2}}{2}}$, so it is necessary to have $\frac{a+\sqrt{a^2-nb^2}}{2}$ is a square in $\mathbb Q$ (and then $d$ is also rational).

We may find better conditions than this one. But I don't know how to manage with the cases $m \ge 3$, because the calculations become difficult. Is there some theoretical approach (e.g. Galois theory) to treat this problem ?

Thank you !

Watson
  • 23,793
  • 6
    You want to compute the prime factorization of $a + b \sqrt{n}$ in a suitable sense. This means understanding the prime ideals of $\mathcal{O}_{\mathbb{Q}(\zeta_n)}$ as well as the ideal class group. For rough results, a necessary but not sufficient condition is that the norm $N(a + b \sqrt{n}) = a^2 - b^2 n$ has an $m^{th}$ root in $\mathbb{Q}$. – Qiaochu Yuan Jan 08 '16 at 22:24
  • 4
    Ah, sorry, when I said "ideal class group" above I meant "unit group," but you might also need to compute the ideal class group. – Qiaochu Yuan Jan 08 '16 at 22:31
  • 2
    Necessary conditions are that the ideal your number generates should be an mth power, and that the number is an nth power modulo all ideals. Local-Global principles should guarantee the converse except in some silly cases of fourth or eighth powers. –  Jan 09 '16 at 13:08
  • Relevant: http://math.stackexchange.com/q/405623 – Watson Dec 24 '16 at 16:24
  • Related: http://math.stackexchange.com/questions/1416720, http://math.stackexchange.com/questions/1008169, http://math.stackexchange.com/questions/1180599, http://math.stackexchange.com/questions/835955, https://math.stackexchange.com/questions/1940784, https://math.stackexchange.com/questions/1910728, https://math.stackexchange.com/questions/196155, https://math.stackexchange.com/questions/1214527, https://math.stackexchange.com/questions/816462, https://math.stackexchange.com/questions/383975, https://math.stackexchange.com/questions/1878085 – Watson Dec 25 '16 at 11:03
  • Related: https://math.stackexchange.com/questions/835955, https://en.wikipedia.org/wiki/Nested_radical, http://www.cybertester.com/data/denest.pdf, http://researcher.watson.ibm.com/researcher/files/us-fagin/symb85.pdf, http://mathworld.wolfram.com/NestedRadical.html, https://www.researchgate.net/publication/256618669_A_note_on_Zippel_Denesting, https://math.stackexchange.com/questions/816462, https://math.stackexchange.com/questions/1097558, https://math.stackexchange.com/questions/1940784 – Watson Dec 25 '16 at 11:10
  • Here is a way to compute more easily the $m$-th roots (or know that they don't exist) : if $a+b\sqrt d = (u+v\sqrt d)^m$, then $N:=a^2-db^2=(u^2-dv^2)^m \implies u^2-dv^2 = (±1)^{m-1} \sqrt[m]{N} =: \lambda$. Then we have $$\sum_{k=0}^m \binom m k u^{m-k} (v\sqrt d)^{k} = a+b\sqrt d.$$ and $u^2 = \lambda +dv^2$ is known (maybe $\lambda$ is only known up to sign when $m$ is even). – Watson Dec 25 '16 at 21:51
  • For instance, when $m=3$ we get $$3u^2+v^3d=b \implies 3(\lambda+v^2d)v+v^3d=b \implies 4dv^3+3\lambda v - b=0$$ For $v=p/q$, the rational root theorem gives $q \mid 4d, p \mid b$ (finitely many possibilites). Try some $p$ and $q$'s and find the corresponding $u$ such that $u^2=\lambda+v^2d$, and check whether $u+v\sqrt d$ is a 3rd root of $a+b\sqrt d$. – Watson Dec 25 '16 at 21:52
  • For $m=4$, we get $$ a = u^4+6u^2v^2d+v^4d^2 \implies 3d^2v^4 + 2\lambda d v^2 + \lambda^2 -a = 0$$ and if $v=p/q$ then $q \mid 3d^2, p \mid (\lambda^2-a)$. Find $v$ and check whether $u$ such that $x=u^2-dv^2$ satisfies $x^4=a+b\sqrt d$. – Watson Dec 25 '16 at 21:54
  • Related: https://math.stackexchange.com/questions/1776159, https://math.stackexchange.com/questions/784496, https://math.stackexchange.com/questions/757917, and https://math.stackexchange.com/questions/394056 – Watson Dec 26 '16 at 12:59
  • Related: https://math.stackexchange.com/questions/2404139/ – Watson Nov 22 '18 at 18:00

3 Answers3

3

The way of Quiaochu Yuan's comment is technical and hard. In elementary way you have with two arbitrary rational $r,s$ form the values given by $a+b\sqrt n=(r+s\sqrt n)^m=A+B\sqrt n$ where $$B=\binom m 1r^{m-1}s+\binom m3r^{m-3}s^3n+\binom m5r^{m-5}s^5n^2+...... $$ and $$A=\text{the other terms}$$ This way you have for each couple of $r,s$ an $A+B\sqrt n$ satisfying the question.

With technical way, you'll have always this implicit condition for some couple of rationals : for each suitable $a+b\sqrt n$ there are two rational $r,s$ fulfilling the elementary condition given here.

Piquito
  • 29,594
3

As suggested by @franz lemmermeyer, a theoretical approach would certainly consist in an adequate global-local principle (i.e. CFT in fine), but there could be technical difficulties when ramification comes into play. Take a general number field $K$, but to avoid petty trouble, assume that the given integer $m$ is odd. The global-local principle for $m$-th powers consists in studying the kernel of the natural map from the global group $ K^* / (K^*)^m$ to the direct sum of all the local groups $K_v^{*} / (K_v^{*})^m $. Given a finite set $S$ of primes of $K$, an element of $K^*$ which is not divisible by any prime $\mathcal L_{v}$ outside $S$ will be called an $S$-unit. The following global-local principle is valid : "an $S$-unit $\alpha$ of $K$ is a global $m$-th power iff for any $\mathcal L_v$ outside $S$, $\alpha$ is an $m$-th power in the local field $K_v$" (Artin-Tate, chap. IX, thm. 1). The finite set $S$ is meant to give us a certain « room » adapted to the problem under study. Here we’ll choose $S$ such that it contains all the infinite primes, all the primes dividing the given $m$ and the given $\alpha$ in $ K^*$,as well as all the primes dividing disc($K$). To decide if $\alpha$ is a global $m$-th power, we have only to look at its natural image in $K_v^* / (K_v^*)^m$ for any $\mathcal L_v$ outside $S$ . Using the Chinese remainder theorem, we can suppose that $m = p^r$ for some rational prime $p$. Let $l$ be the rational prime under such an $\mathcal L_v$ . By our choice of $S$, $l \neq p$, $K_v$ is an unramified extension of $\mathbf Q_l$, and $\alpha \in U_v$, the group of units of $K_v$. Let $\kappa_v$ be the residual field of $K_v$, a finite field of degree over $\mathbf F_l$ equal to the inertia index, equal here to the local degree $[K_v : \mathbf Q_l]$. It is classically known that $U_v$ is the direct product of a group $\cong(\kappa_{v})^*$ (via Hensel’s lemma) and of the group of principal units $U_1 = 1 + \mathcal L_v$ . Since $l \neq p$, raising to a $p$-primary power is an automorphism of $U_1$, hence in the end $ U_v / (U_v)^{p^r} \cong \kappa_v^* / (\kappa_v^*)^{p^r}$.

Let us now switch to the case at hand, where $K$ is a quadratic field. We have only to consider two cases : either $l$ is inert in $K$, or $l$ is split. In the first case, $\kappa_l^*$ is cyclic of order $l^2 – 1$ ; in the second, $\kappa_v$ cyclic of order $l – 1$ for any of the two $v$’s above $l$. Define $W_r (l)$ to be the quotient $\kappa_l^*$ mod $p^r$ or the product of the two quotients $\kappa_v^*$ mod $p^r$ . We know explicitly $W_r (l)$ (without feeling like writing it down !), and the conclusion is : let $\alpha \in K^* $; choose $S$ as above ; then $\alpha$ is a $p^r$-th power in $K^*$ iff for any $l$ outside $S$, the natural image of $\alpha$ in $W_r(l)$ is trivial.

  • Thank you very much for your answer! What do you mean by "local group" at the beginning? In particular, what does $K_{\nu}$ mean? I do not know so much about local fields… – Watson May 10 '16 at 20:18
  • 2
    Given a prime ideal P_v of a global field K, one can associate to it a P_v-adic valuation v, which in turn gives an ultrametric distance, and K_v denotes the completion of K w.r.t. this distance. This K_v is usually called a P_v local field, "local" as opposed to "global". The point is that the arithmetic of a local field is much simpler, in particular its ring of integers has only one maximal ideal (more precisely, it's a discrete valuation ring). The so called local - global principle "phlosophically" means that the knowledge of (some) global properties should come from that of all the ... – nguyen quang do May 11 '16 at 06:01
  • 2
    ... corresponding local ones. Some classical examples : - the theory of quadratic forms over a number field - Hasse's norm theorem for cyclic extensions - the determination of the Brauer group - the two latter examples are part of CFT from a local-global point of view . An even more far reaching illustration is the current theory of "motifs" (initiated by Grothendieck) which, in some way, "explains" how the archimedean and p-adic worlds can be related, although they are opposed by nature (connected vs. totally disconnected) . – nguyen quang do May 11 '16 at 06:10
  • Thank you for these helpful explanations! The $P_v$-adic valuation takes an ideal $I$ of the ring of integers $\mathcal O_K$ and returns the number of times $P_v$ appears in the decomposition of $I$ into prime ideals of $\mathcal O_K$, is that right? $\tag*{}$ So the ultrametric distance it induces is defined on the set of the ideals of $\mathcal O_K$, or am I wrong? This is because I don't understand how you can complete $;K;$… – Watson May 11 '16 at 08:20
  • 2
    No, the P_v -adic additive valuation is defined as a map from K* to Z by letting v(x) = the exponent of P_v in the principal ideal (x). You can transform this into a (multiplicative) norm by defining the "absolute value" of x as being p^{-v(x)}, where p is the rational prime under P_v (there is a choice of normalization here, but it does not affect the topology). The completion of K is taken w.r.t. this absolute value.You can find all this in any (advanced) text book on ANT, such as the first two chapters of the classical Cassels-Fröhlich's book. – nguyen quang do May 11 '16 at 10:21
  • When you are writing «$S$ [is a finite set] of primes of $K$», you mean that $S$ is a set of prime ideals of $\mathcal O_K$ ? $\tag{}$ 2. What do you mean by a prime $\mathcal L_{v}$ ? $\tag{}$ 3. What do you mean by «the infinite primes» ? Sorry for these "silly" questions. Maybe you could edit your answer if it is too long for a comment. Thank you anyway!
  • – Watson May 11 '16 at 19:21
  • 2
  • Yes 2. Just a prime ideal, indexed by the letter v which will denote the associated valuation 3. In the usual language of ANT, the "infinite primes" of K are the absolute values induced by the embeddings of K into an algebraic closure of Q. NB: we are suggested by the moderator to move our discussion to "chat"
  • – nguyen quang do May 11 '16 at 19:37
  • I would have two other questions on http://chat.stackexchange.com/rooms/39643, 4. is solved, but 5. and 6. are unsolved for me. Let me know if you are bothered by my several questions. But I would really enjoy understanding your interesting answer. – Watson May 12 '16 at 11:34